Stay ahead of learning milestones! Enroll in a class over the summer!

G
Topic
First Poster
Last Poster
IMC 2018 P1
ThE-dArK-lOrD   3
N 3 hours ago by Fibonacci_math
Source: IMC 2018 P1
Let $(a_n)_{n=1}^{\infty}$ and $(b_n)_{n=1}^{\infty}$ be two sequences of positive numbers. Show that the following statements are equivalent:
[list=1]
[*]There is a sequence $(c_n)_{n=1}^{\infty}$ of positive numbers such that $\sum_{n=1}^{\infty}{\frac{a_n}{c_n}}$ and $\sum_{n=1}^{\infty}{\frac{c_n}{b_n}}$ both converge;[/*]
[*]$\sum_{n=1}^{\infty}{\sqrt{\frac{a_n}{b_n}}}$ converges.[/*]
[/list]

Proposed by Tomáš Bárta, Charles University, Prague
3 replies
ThE-dArK-lOrD
Jul 24, 2018
Fibonacci_math
3 hours ago
Infinite series involving tau function
bakkune   1
N 3 hours ago by Safal
For each positive integer $n$, let $\tau(n)$ be the number of positive divisors of $n$. Evaluate
$$
\sum_{n=1}^{+\infty} (-1)^n \frac{\tau(n)}{n}
$$
1 reply
bakkune
Today at 4:35 AM
Safal
3 hours ago
two solutions
τρικλινο   4
N 5 hours ago by Safal
in a book:CORE MATHS for A-LEVEL ON PAGE 41 i found the following


1st solution


$x^2-5x=0$



$ x(x-5)=0$



hence x=0 or x=5



2nd solution



$x^2-5x=0$

$x-5=0$ dividing by x



hence the solution x=0 has been lost



is the above correct?
4 replies
τρικλινο
Yesterday at 6:20 PM
Safal
5 hours ago
high school math
aothatday   5
N Today at 3:32 AM by aothatday
Let $x_n$ be a positive root of the equation $x_n^n=x^2+x+1$. Prove that the following sequence converges: $n^2(x_n-x_{ n+1})$
5 replies
aothatday
Apr 10, 2025
aothatday
Today at 3:32 AM
Putnam 2003 B1
btilm305   13
N Today at 12:08 AM by clarkculus
Do there exist polynomials $a(x)$, $b(x)$, $c(y)$, $d(y)$ such that \[1 + xy + x^2y^2= a(x)c(y) + b(x)d(y)\] holds identically?
13 replies
btilm305
Jun 23, 2011
clarkculus
Today at 12:08 AM
High School Integration Extravaganza Problem Set
Riemann123   12
N Yesterday at 5:20 PM by jkim0656
Source: River Hill High School Spring Integration Bee
Hello AoPS!

Along with user geodash2, I have organized another high-school integration bee (River Hill High School Spring Integration Bee) and wanted to share the problems!

We had enough folks for two concurrent rooms, hence the two sets. (ARML kids from across the county came.)

Keep in mind that these integrals were written for a high-school contest-math audience. I hope you find them enjoyable and insightful; enjoy!


[center]Warm Up Problems[/center]
\[
\int_{1}^{2} \frac{x^{3}+x^2}{x^5}dx
\]\[\int_{2025}^{2025^{2025}}\frac{1}{\ln\left(2025\right)\cdot x}dx\]\[
\int(\sin^2(x)+\cos^2(x)+\sec^2(x)+\csc^2(x))dx
\]\[
\int_{-2025.2025}^{2025.2025}\sin^{2025}(2025x)\cos^{2025}(2025x)dx
\]\[
    \int_{\frac \pi 6}^{\frac \pi 3} \tan(\theta)^2d\theta
\]\[
\int  \frac{1+\sqrt{t}}{1+t}dt
\]-----
[center]Easier Division Set 1[/center]
\[\int \frac{x^{2}+2x+1}{x^{3}+3x^{2}+3x+3}dx
\]\[\int_{0}^{\frac{3\pi}{2}}\left(\frac{\pi}{2}-x\right)\sin\left(x\right)dx\]\[
\int_{-\pi/2}^{\pi/2}x^3e^{-x^2}\cos(x^2)\sin^2(x)dx
\]\[
\int\frac{1}{\sqrt{12-t^{2}+4t}}dt
\]\[
\int \frac{\sqrt{e^{8x}}}{e^{8x}-1}dx
\]-----
[center]Easier Division Set 2[/center]
\[
\int \frac{e^x}{e^{2x}+1} dx
\]\[
\int_{-5}^5\sqrt{25-u^2}du
\]\[
\int_{-\frac12}^\frac121+x+x^2+x^3\ldots dx
\]\[\int \cos(\cos(\cos(\ln \theta)))\sin(\cos(\ln \theta))\sin(\ln \theta)\frac{1}{\theta}d\theta\]\[\int_{0}^{\frac{1}{6}}\frac{8^{2x}}{64^{2x}-8^{\left(2x+\frac{1}{3}\right)}+2}dx\]-----
[center]Harder Division Set 1[/center]
\[\int_{0}^{\frac{\pi}{2}}\frac{\sin\left(x\right)}{\sin\left(x\right)+\cos\left(x\right)}+\frac{\sin\left(\frac{\pi}{2}-x\right)}{\sin\left(\frac{\pi}{2}-x\right)+\cos\left(\frac{\pi}{2}-x\right)}dx\]\[
\int_0^{\infty}e^{-x}\Bigl(\cos(20x)+\sin(20x)\Bigr) dx
\]\[
\lim_{n\to \infty}\frac{1}{n}\int_{1}^{n}\sin(nt)^2dt
\]\[
\int_{x=0}^{x=1}\left( \int_{y=-x}^{y=x} \frac{y^2}{x^2+y^2}dy\right)dx
\]\[
\int_{0}^{13}\left\lceil\log_{10}\left(2^{\lceil x\rceil }x\right)\right\rceil dx
\]-----
[center]Harder Division Set 2[/center]
\[
\int \frac{6x^2}{x^6+2x^3+2}dx
\]\[
\int -\sin(2\theta)\cos(\theta)d\theta
\]\[
\int_{0}^{5}\sin(\frac{\pi}2 \lfloor{x}\rfloor x) dx
\]\[
\int_{0}^{1} \frac{\sin^{-1}(\sqrt{x})^2}{\sqrt{x-x^2}}dx
\]\[
\int\left(\cot(\theta)+\tan(\theta)\right)^2\cot(2\theta)^{100}d\theta
\]-----
[center]Bonanza Round (ie Fun/Hard/Weird Problems) (In No Particular Order)[/center]
\[
\int \ln\left\{\sqrt[7]{x}^\frac1{\ln\left\{\sqrt[5]{x}^\frac1{\ln\left\{\sqrt[3]{x}^\frac1{\ln\left\{\sqrt{x}\right\}}\right\}}\right\}}\right\}dx
\]\[\int_{1}^{{e}^{\pi}} \cos(\ln(\sqrt{u}))du\]\[
\int_e^{\infty}\frac {1-x\ln{x}}{xe^x}dx
\]\[\int_{0}^{1}\frac{e^{x}}{\left(x^{2}+3x+2\right)^{\frac{1}{2^{1}}}}\times\frac{e^{-\frac{x^{2}}{2}}}{\left(x^{2}+3x+2\right)^{\frac{1}{2^{2}}}}\times\frac{e^{\frac{x^{3}}{3}}}{\left(x^{2}+3x+2\right)^{\frac{1}{2^{3}}}}\times\frac{e^{-\frac{x^{4}}{4}}}{\left(x^{2}+3x+2\right)^{\frac{1}{2^{4}}}} \ldots \,dx\]
For $x$ on the domain $-0.2025\leq x\leq 0.2025$ it is known that \[\displaystyle f(x)=\sin\left(\int_{0}^x \sqrt[3]{\cos\left(\frac{\pi}{2} t\right)^3+26}\ dt\right)\]is invertible. What is $\displaystyle (f^{-1})'(0)$?
12 replies
Riemann123
Friday at 2:11 PM
jkim0656
Yesterday at 5:20 PM
limiting behavior of the generalization of IMO 1968/6 for arbitrary powers
revol_ufiaw   1
N Yesterday at 3:17 PM by alexheinis
Source: inspired by IMO 1968/6
Define $f : \mathbb{N} \rightarrow \mathbb{N}$ by
\[f(n) = \sum_{i\ge 0} \bigg\lfloor \frac{n + a^i}{a^{i+1}}\bigg\rfloor=\bigg\lfloor \frac{n + 1}{a} \bigg\rfloor + \bigg\lfloor \frac{n + a}{a^2} \bigg\rfloor + \bigg\lfloor \frac{n + a^2}{a^3} \bigg\rfloor + \cdots\]for some fixed $a \in \mathbb{N}$. Prove that
\[\lim_{n \rightarrow \infty} \frac{f(n)}{n/(a-1)} = 1.\]
[P.S.: IMO 1968/6 asks to prove $f(n) = n$ for $a = 2$.]
1 reply
revol_ufiaw
Yesterday at 1:43 PM
alexheinis
Yesterday at 3:17 PM
Putnam 2015 B4
Kent Merryfield   22
N Yesterday at 2:58 PM by lpieleanu
Let $T$ be the set of all triples $(a,b,c)$ of positive integers for which there exist triangles with side lengths $a,b,c.$ Express \[\sum_{(a,b,c)\in T}\frac{2^a}{3^b5^c}\]as a rational number in lowest terms.
22 replies
Kent Merryfield
Dec 6, 2015
lpieleanu
Yesterday at 2:58 PM
A real analysis Problem from contest
Safal   2
N Yesterday at 11:17 AM by Safal
Source: Random.
Let $f: (0,\infty)\rightarrow \mathbb{R}$ be a function such that $$\lim_{x\rightarrow \infty} f(x)=1$$and $$f(x+1)=f(x)$$for all $x\in (0,\infty)$

Prove or disprove the following statements.

1.$f$ is continuous.
2.$f$ is bounded.

Is My Idea correct?
2 replies
Safal
Yesterday at 8:34 AM
Safal
Yesterday at 11:17 AM
maximal determinant
EthanWYX2009   4
N Yesterday at 10:59 AM by loup blanc
Source: 2023 Aug taca-9
Let matrix
\[A=\begin{bmatrix} 1&1&1&1&1\\1&-1&1&-1&1\\?&?&?&?&?\\?&?&?&?&?\\?&?&?&?&?\end{bmatrix}\in\mathbb R^{5\times 5}\]satisfy $\text{tr} (AA^T)=28.$ Determine the maximum value of $\det A.$
4 replies
EthanWYX2009
Apr 9, 2025
loup blanc
Yesterday at 10:59 AM
Two circles are tangents in a triangles with angle 60
nAalniaOMliO   1
N Mar 29, 2025 by sunken rock
Source: Belarusian National Olympiad 2025
In a triangle $ABC$ angle $\angle BAC = 60^{\circ}$. Point $M$ is the midpoint of $BC$, and $D$ is the foot of altitude from point $A$. Points $T$ and $P$ are marked such that $TBC$ is equilateral, and $\angle BPD=\angle DPC = 30^{\circ}$ and this points lie in the same half-plane with respect to $BC$, not in the same as $A$.
Prove that the circumcircles of $ADP$ and $AMT$ are tangent.
Ivan Korshunau
1 reply
nAalniaOMliO
Mar 28, 2025
sunken rock
Mar 29, 2025
Two circles are tangents in a triangles with angle 60
G H J
G H BBookmark kLocked kLocked NReply
Source: Belarusian National Olympiad 2025
The post below has been deleted. Click to close.
This post has been deleted. Click here to see post.
nAalniaOMliO
295 posts
#1
Y by
In a triangle $ABC$ angle $\angle BAC = 60^{\circ}$. Point $M$ is the midpoint of $BC$, and $D$ is the foot of altitude from point $A$. Points $T$ and $P$ are marked such that $TBC$ is equilateral, and $\angle BPD=\angle DPC = 30^{\circ}$ and this points lie in the same half-plane with respect to $BC$, not in the same as $A$.
Prove that the circumcircles of $ADP$ and $AMT$ are tangent.
Ivan Korshunau
This post has been edited 2 times. Last edited by nAalniaOMliO, Apr 4, 2025, 4:47 PM
Z K Y
The post below has been deleted. Click to close.
This post has been deleted. Click here to see post.
sunken rock
4381 posts
#2
Y by
Observation only: $T$ is circumcenter of $\triangle BPD$, I do not know if it will be helpful.
Z K Y
N Quick Reply
G
H
=
a